K
Khách

Hãy nhập câu hỏi của bạn vào đây, nếu là tài khoản VIP, bạn sẽ được ưu tiên trả lời.

3 tháng 6 2019

Đầu tiên ta chứng minh: \(\left(a+b+c\right)\left(x+y+z\right)\le3\left(ax+by+cz\right)\)

\(\Leftrightarrow ay+az+bz+bx+cx+cy\le2\left(ax+by+cz\right)\)

\(\Leftrightarrow a\left(y+z-2x\right)+b\left(z+x-2y\right)+c\left(x+y-2z\right)\le0\)

\(\Leftrightarrow a\left(y+z-2x\right)-b\left[\left(y+z-2x\right)+\left(x+y-2z\right)\right]+c\left(x+y-2z\right)\le0\)

\(\Leftrightarrow\left(a-b\right)\left(y+z-2x\right)+\left(c-b\right)\left(x+y-2z\right)\le0\)

Không mất tính tổng quát, giả sử: \(\hept{\begin{cases}a\ge b\ge c\\x\ge y\ge z\end{cases}}\)

Theo đó: \(\hept{\begin{cases}a-b\ge0\\y+z-2x\le0\end{cases}}\Rightarrow\left(a-b\right)\left(y+z-2x\right)\le0\)

Tương tự \(\left(c-b\right)\left(x+y-2z\right)\le0\)

Ta có đpcm.

Áp dụng vào bài toán:

Đặt \(a^2+b^2=x;b^2+c^2=y;c^2+a^2=z;a+b=p;b+c=q;c+a=o\), ta có:

Đpcm \(\Leftrightarrow\frac{x}{p}+\frac{y}{q}+\frac{z}{o}\le\frac{3\cdot\frac{1}{2}\left(x+y+z\right)}{\frac{1}{2}\left(p+q+o\right)}=\frac{3\left(x+y+z\right)}{p+q+o}\)

\(\Leftrightarrow\left(\frac{x}{p}+\frac{y}{q}+\frac{z}{o}\right)\left(p+q+o\right)\le3\left(x+y+z\right)\)[*]

Mà theo bất đẳng thức đã chứng minh:

\(VT\left[+\right]\le3\left(\frac{x}{p}\cdot p+\frac{y}{q}\cdot q+\frac{z}{o}\cdot o\right)=3\left(x+y+z\right)=VP\)

Ta có đpcm

Dấu "=" xảy ra khi a = b = c

3 tháng 6 2019

Câu hỏi của Lưu Hải Dương - Toán lớp 9 | Học trực tuyến

23 tháng 7 2020

Cô Quản Lý Nguyễn Linh Chi ơi cô bảo bạn đăng bài tham khảo bạn làm nhưng đã có ai làm bài đâu ạ 

23 tháng 7 2020

\(\frac{a^2+b^2}{a+b}+\frac{b^2+c^2}{b+c}+\frac{c^2+a^2}{c+a}\le\frac{3\left(a^2+b^2+c^2\right)}{a+b+c}\)

\(\Leftrightarrow\left(a+b+c\right)\left(\frac{a^2+b^2}{a+b}+\frac{b^2+c^2}{b+c}+\frac{c^2+a^2}{c+a}\right)\le3\left(a^2+b^2+c^2\right)\)

\(\Leftrightarrow\frac{c\left(a^2+b^2\right)}{a+b}+\frac{a\left(b^2+c^2\right)}{b+c}+\frac{b\left(c^2+a^2\right)}{c+a}\le a^2+b^2+c^2\)

\(\Leftrightarrow\left(\frac{\left(a^2+b^2\right)c}{a+b}-c^2\right)+\left(\frac{\left(b^2+c^2\right)a}{b+c}-a^2\right)+\left(\frac{\left(c^2+a^2\right)b}{c+a}-b^2\right)\)

\(\Leftrightarrow\frac{ac\left(a-c\right)+bc\left(b-c\right)}{a+b}+\frac{ab\left(b-a\right)+ca\left(c-a\right)}{b+c}\)

\(+\frac{bc\left(c-b\right)+ab\left(a-b\right)}{c+a}\le0\)

\(\Leftrightarrow ab\left(a-b\right)\left(\frac{1}{c+a}-\frac{1}{b+c}\right)+ca\left(c-a\right)\left(\frac{1}{b+c}-\frac{1}{a+b}\right)\)

\(+bc\left(b-c\right)\left(\frac{1}{a+b}-\frac{1}{a+c}\right)\le0\)

\(\Leftrightarrow\frac{-ac\left(c-a\right)^2}{\left(a+b\right)\left(b+c\right)}+\frac{-bc\left(c-b\right)^2}{\left(a+b\right)\left(a+c\right)}+\frac{-ab\left(b-a\right)^2}{\left(a+c\right)\left(b+c\right)}\le0\)*đúng với mọi a,b,c dương*

Vậy bất đẳng thức được chứng minh

Đẳng thức xảy ra khi a = b = c

1 tháng 1 2021

giả sử \(a\ge b\ge c\ge0\)

Ta có: \(a+\frac{b}{2}-\frac{a^2+ab+b^2}{a+b}=\frac{1}{2}\left(ab-b^2\right)\ge0\Rightarrow a+\frac{b}{2}\ge\frac{a^2+ab+b^2}{a+b}\)

\(b+\frac{a}{2}-\frac{a^2+ab+b^2}{a+b}=\frac{1}{2}\left(ab-a^2\right)\le0\Rightarrow b+\frac{a}{2}\le\frac{a^2+ab+b^2}{a+b}\)

Tương tự: \(b+\frac{c}{2}\ge\frac{b^2+bc+c^2}{b+c}\ge c+\frac{b}{2};a+\frac{c}{2}\ge\frac{a^2+ac+c^2}{a+c}\ge c+\frac{a}{2}\)

Lại có:+) \(\frac{a^3-b^3}{a+b}+\frac{b^3-c^3}{b+c}+\frac{c^3-a^3}{c+a}\)

\(=\left(a-b\right)\frac{a^2+ab+b^2}{a+b}+\left(b-c\right)\frac{b^2+bc+c^2}{b+c}-\left(a-c\right)\frac{a^2+ac+c^2}{a+c}\)

\(\ge\left(a-b\right)\left(b+\frac{a}{2}\right)+\left(b-c\right)\left(c+\frac{a}{2}\right)-\left(a-c\right)\left(a+\frac{c}{2}\right)\)

\(\ge\frac{-1}{4}\left[\left(a-b\right)^2+\left(b-c\right)^2+\left(c-a\right)^2\right]\left(1\right)\)

+) \(\frac{a^3-b^3}{a+b}+\frac{b^3-c^3}{b+c}+\frac{c^3-a^3}{c+a}\)

\(=\left(a-b\right)\frac{a^2+ab+b^2}{a+b}+\left(b-c\right)\frac{b^2+bc+c^2}{b+c}-\left(a-c\right)\frac{a^2+ac+c^2}{a+c}\)

\(\le\left(a-b\right)\left(a+\frac{b}{2}\right)+\left(b-c\right)\left(b+\frac{c}{2}\right)-\left(a-c\right)\left(c+\frac{a}{2}\right)\)

\(\le\frac{1}{4}\left[\left(a-b\right)^2+\left(b-c\right)^2+\left(c-a\right)^2\right]\left(2\right)\)

Từ 1,2 => đpcm

2 tháng 1 2021

BĐT đã cho tuong duong voi:

\(\left|\frac{\left(a-b\right)\left(b-c\right)\left(c-a\right)\left(ab+bc+ca\right)}{\left(a+b\right)\left(b+c\right)\left(c+a\right)}\right|\le\frac{1}{4}\left[\Sigma\left(a-b\right)^2\right]\)

Theo AM-GM ta có: \(\left(ab+bc+ca\right)\le\frac{9}{8}\cdot\frac{\left(a+b\right)\left(b+c\right)\left(c+a\right)}{a+b+c}\)

Có: \(VT\le\frac{9}{8}\left|\frac{\sqrt{\left(a-b\right)^2\left(b-c\right)^2\left(c-a\right)^2}}{\left(a+b+c\right)}\right|=\frac{9\sqrt{\left(a-b\right)^2\left(b-c\right)^2\left(c-a\right)^2}}{8\left(a+b+c\right)}\)

Cần chứng minh: \(4\left(a+b+c\right)^2\left[\Sigma\left(a-b\right)^2\right]^2\ge9\left(a-b\right)^2\left(b-c\right)^2\left(c-a\right)^2\)

Rõ ràng \(\Sigma\left(a-b\right)^2\ge3\sqrt[3]{\left(a-b\right)^2\left(b-c\right)^2\left(c-a\right)^2}\)

Cần cm: \(36\left(a+b+c\right)^2\sqrt[3]{\left(a-b\right)^4\left(b-c\right)^4\left(c-a\right)^4}\ge9\sqrt[3]{\left(a-b\right)^6\left(b-c\right)^6\left(c-a\right)^6}\)

Hay \(4\left(a+b+c\right)^2\ge\sqrt[3]{\left(a-b\right)^2\left(b-c\right)^2\left(c-a\right)^2}\)

Tiếp tục là điều hiển nhiên do \(VT\ge4\left[\left(a+b+c\right)^2-3\left(ab+bc+ca\right)\right]\)

\(=2\left[\left(a-b\right)^2+\left(b-c\right)^2+\left(c-a\right)^2\right]\)

\(\ge6\sqrt[3]{\left(a-b\right)^2\left(b-c\right)^2\left(c-a\right)^2}\ge VP\)

Đẳng thức xảy ra khi \(\hept{\begin{cases}\left(a-b\right)\left(b-c\right)\left(c-a\right)=0\\a-b=b-c=c-a\\a=b=c\end{cases}}\Leftrightarrow a=b=c.\)

2 tháng 12 2020

Đặt \(\left(\frac{1}{a},\frac{1}{b},\frac{1}{c}\right)=\left(x,y,z\right)\)

\(x+y+z\ge\frac{x^2+2xy}{2x+y}+\frac{y^2+2yz}{2y+z}+\frac{z^2+2zx}{2z+x}\)

\(\Leftrightarrow x+y+z\ge\frac{3xy}{2x+y}+\frac{3yz}{2y+z}+\frac{3zx}{2z+x}\)

\(\frac{3xy}{2x+y}\le\frac{3}{9}xy\left(\frac{1}{x}+\frac{1}{x}+\frac{1}{y}\right)=\frac{1}{3}\left(x+2y\right)\)

\(\Rightarrow\Sigma_{cyc}\frac{3xy}{2x+y}\le\frac{1}{3}\left[\left(x+2y\right)+\left(y+2z\right)+\left(z+2x\right)\right]=x+y+z\)

Dấu "=" xảy ra khi x=y=z

NV
10 tháng 11 2019

\(P=\sum\frac{a}{\sqrt{\left(2a\right)^2+\left(b+c\right)^2}}\le\sqrt{2}\sum\frac{a}{2a+b+c}=\sqrt{2}\sum a\left(\frac{1}{a+b+a+c}\right)\le\frac{\sqrt{2}}{4}\sum\left(\frac{a}{a+b}+\frac{a}{a+c}\right)=\frac{3\sqrt{2}}{4}\)

Dấu "=" xảy ra khi \(a=b=c\)

7 tháng 5 2020

\(\Leftrightarrow\Sigma\sqrt{\frac{3a^3}{\left[5a^2+\left(b+c\right)^2\right]\left(a+b+c\right)}}\le1\)

Theo Am-GM: \(VT=\Sigma\sqrt{\frac{3a^2}{5a^2+\left(b+c\right)^2}.\frac{a}{a+b+c}}\le\Sigma\frac{3a^2}{2\left(5a^2+\left(b+c\right)^2\right)}+\frac{1}{2}\)

Như vậy nó là đủ để chứng minh rằng: \(\Sigma\frac{3a^2}{5a^2+\left(b+c\right)^2}\le1\)

Giả sử \(c=min\left\{a,b,c\right\}\) nó tương đương:

$$2\, \left( a-b \right) ^{2} \left( 3\,c+a+b \right) \left( -c+a+b
 \right) \left( {a}^{2}+2\,ab+{b}^{2}+5\,{c}^{2} \right) +2\,c
 \left( a-c \right) \left( b-c \right) \left( 3\,{a}^{3}+9\,{a}^{2}b
+17\,c{a}^{2}+9\,a{b}^{2}-20\,abc+3\,{c}^{2}a+3\,{b}^{3}+17\,c{b}^{2}+
3\,{c}^{2}b+{c}^{3} \right) \geqq 0$$

(Gõ Latex, không hiện thì vô thống kê hỏi đáp xem)

Đây là điều hiển nhiên/

PS: Bài này quan trọng là ý tưởng phá căn thôi chứ không có gì khó. Lúc đầu UCT bất đẳng thức cuối cho đẹp nhưng phải xét các TH mệt lắm, chưa rành nên không làm cách đó:D

7 tháng 5 2020

Chứng minh: \(\Sigma\frac{3a^2}{5a^2+\left(b+c\right)^2}\le1\), cách 2:

Đổi biến sang pqr: (Vô thống kê hỏi đáp xem nếu olm không hiện Latex)

Nếu \(p^2\le4q\) ta cần:

$$2/9\,p \left( 19\,{p}^{2}-36\,q \right) \left( {p}^{3}-4\,qp+9\,r
 \right) -4/9\, \left( {p}^{2}-3\,q \right) \left( {p}^{2}-4\,q
 \right) \left( 5\,{p}^{2}-3\,q \right) \geqq 0$$

(Hiển nhiên)

Nếu \(p^2\ge4q\) thì cần chứng minh:

$$2\,p \left( 19\,{p}^{2}-36\,q \right) r+2\, \left( {p}^{2}-4\,q
 \right) \left( {p}^{4}-2\,{q}^{2} \right) \geqq 0$$

(Hiển nhiên)

Từ 2 TH trên ta thu được điều phải chứng minh.

Giúp mình với! Mình đang cần gấp. Các bạn làm được bài nào thì giúp đỡ mình nhé! Cảm ơn!Bài 1: Cho các số thực dương a,b,c. Chứng minh rằng:\(\frac{a^2}{\sqrt{\left(2a^2+b^2\right)\left(2a^2+c^2\right)}}+\frac{b^2}{\sqrt{\left(2b^2+c^2\right)\left(2b^2+a^2\right)}}+\frac{c^2}{\sqrt{\left(2c^2+a^2\right)\left(2c^2+b^2\right)}}\le1\).Bài 2: Cho các số thực dương a,b,c,d. Chứng minh...
Đọc tiếp

Giúp mình với! Mình đang cần gấp. Các bạn làm được bài nào thì giúp đỡ mình nhé! Cảm ơn!

Bài 1: Cho các số thực dương a,b,c. Chứng minh rằng:

\(\frac{a^2}{\sqrt{\left(2a^2+b^2\right)\left(2a^2+c^2\right)}}+\frac{b^2}{\sqrt{\left(2b^2+c^2\right)\left(2b^2+a^2\right)}}+\frac{c^2}{\sqrt{\left(2c^2+a^2\right)\left(2c^2+b^2\right)}}\le1\).

Bài 2: Cho các số thực dương a,b,c,d. Chứng minh rằng:

\(\frac{a-b}{a+2b+c}+\frac{b-c}{b+2c+d}+\frac{c-d}{c+2d+a}+\frac{d-a}{d+2a+b}\ge0\).

Bài 3: Cho các số thực dương a,b,c. Chứng minh rằng:

\(\frac{\sqrt{b+c}}{a}+\frac{\sqrt{c+a}}{b}+\frac{\sqrt{a+b}}{c}\ge\frac{4\left(a+b+c\right)}{\sqrt{\left(a+b\right)\left(b+c\right)\left(c+a\right)}}\).

Bài 4:Cho a,b,c>0, a+b+c=3. Chứng minh rằng: 

a)\(\frac{a^3}{a^2+ab+b^2}+\frac{b^3}{b^2+bc+c^2}+\frac{c^3}{c^2+ca+a^2}\ge1\).

b)\(\frac{a^3}{a^2+b^2}+\frac{b^3}{b^2+c^2}+\frac{c^3}{c^2+a^2}\ge\frac{3}{2}\).

c)\(\frac{a+1}{b^2+1}+\frac{b+1}{c^2+1}+\frac{c+1}{a^2+1}\ge3\).

Bài 5: Cho a,b,c >0. Chứng minh rằng:

\(\frac{2a^2+ab}{\left(b+c+\sqrt{ca}\right)^2}+\frac{2b^2+bc}{\left(c+a+\sqrt{ab}\right)^2}+\frac{2c^2+ca}{\left(a+b+\sqrt{bc}\right)^2}\ge1\).

8
21 tháng 10 2019

1) Áp dụng bunhiacopxki ta được \(\sqrt{\left(2a^2+b^2\right)\left(2a^2+c^2\right)}\ge\sqrt{\left(2a^2+bc\right)^2}=2a^2+bc\), tương tự với các mẫu ta được vế trái \(\le\frac{a^2}{2a^2+bc}+\frac{b^2}{2b^2+ac}+\frac{c^2}{2c^2+ab}\le1< =>\)\(1-\frac{bc}{2a^2+bc}+1-\frac{ac}{2b^2+ac}+1-\frac{ab}{2c^2+ab}\le2< =>\)

\(\frac{bc}{2a^2+bc}+\frac{ac}{2b^2+ac}+\frac{ab}{2c^2+ab}\ge1\)<=> \(\frac{b^2c^2}{2a^2bc+b^2c^2}+\frac{a^2c^2}{2b^2ac+a^2c^2}+\frac{a^2b^2}{2c^2ab+a^2b^2}\ge1\)  (1) 

áp dụng (x2 +y2 +z2)(m2+n2+p2\(\ge\left(xm+yn+zp\right)^2\)

(2a2bc +b2c2 + 2b2ac+a2c2 + 2c2ab+a2b2). VT\(\ge\left(bc+ca+ab\right)^2\)   <=> (ab+bc+ca)2. VT \(\ge\left(ab+bc+ca\right)^2< =>VT\ge1\)  ( vậy (1) đúng)

dấu '=' khi a=b=c

21 tháng 10 2019

4b, \(\frac{a^3}{a^2+b^2}+\frac{b^3}{b^2+c^2}+\frac{c^3}{c^2+a^2}=1-\frac{ab^2}{a^2+b^2}+1-\frac{bc^2}{b^2+c^2}+1-\frac{ca^2}{a^2+c^2}\)

\(\ge3-\frac{ab^2}{2ab}-\frac{bc^2}{2bc}-\frac{ca^2}{2ac}=3-\frac{\left(a+b+c\right)}{2}=\frac{3}{2}\)